Last visit was: 26 Apr 2024, 06:44 It is currently 26 Apr 2024, 06:44

Close
GMAT Club Daily Prep
Thank you for using the timer - this advanced tool can estimate your performance and suggest more practice questions. We have subscribed you to Daily Prep Questions via email.

Customized
for You

we will pick new questions that match your level based on your Timer History

Track
Your Progress

every week, we’ll send you an estimated GMAT score based on your performance

Practice
Pays

we will pick new questions that match your level based on your Timer History
Not interested in getting valuable practice questions and articles delivered to your email? No problem, unsubscribe here.
Close
Request Expert Reply
Confirm Cancel
User avatar
Manager
Manager
Joined: 14 Jun 2009
Posts: 53
Own Kudos [?]: 80 [0]
Given Kudos: 9
 Q47  V35
Send PM
User avatar
Manager
Manager
Joined: 12 Aug 2008
Posts: 177
Own Kudos [?]: 17 [0]
Given Kudos: 1
Location: Buffalo, NY
Concentration: Accounting/Finance
Schools:Simon '11
 Q48  V47
Send PM
User avatar
Senior Manager
Senior Manager
Joined: 27 Jun 2008
Posts: 329
Own Kudos [?]: 136 [0]
Given Kudos: 92
Concentration: General Management
WE 1: Investment Banking - 6yrs
Send PM
avatar
Intern
Intern
Joined: 15 May 2009
Posts: 7
Own Kudos [?]: 1 [0]
Given Kudos: 0
Send PM
Re: There is relativley little room for growth in the overall [#permalink]
B for me.
User avatar
Manager
Manager
Joined: 15 Jan 2008
Posts: 131
Own Kudos [?]: 65 [2]
Given Kudos: 3
Send PM
Re: There is relativley little room for growth in the overall [#permalink]
2
Kudos
The question asks to find out a premise which casts most doubt on the conclusion that the only way to increase the market share is to buy competitors...

But the option B supports the conclusion that buying the competitors.

BUT ISNT this wrong.

I will go with option D as it suggests that there are other ways to capture the market share and that way is price reductions.
User avatar
Manager
Manager
Joined: 14 Jun 2009
Posts: 53
Own Kudos [?]: 80 [0]
Given Kudos: 9
 Q47  V35
Send PM
Re: There is relativley little room for growth in the overall [#permalink]
i think ur right...this is what was confusing me...thanks a tonne...option D for me :)
User avatar
Director
Director
Joined: 03 Jun 2009
Posts: 578
Own Kudos [?]: 2326 [0]
Given Kudos: 56
Location: New Delhi
Concentration: IT Consultancy
 Q50  V38
WE 1: 5.5 yrs in IT
Send PM
Re: There is relativley little room for growth in the overall [#permalink]
B is strengthening the argument, whereas we have been asked to select an option which weakens the argument.

Correct answer is D

(A) Most of the major carpet producers market other floor coverings as well. -Out of scope
(B) Most established carpet producers market several different brand names and varieties, and there is no remaining niche in the market for new brands to fill. -by saying "there is no remaining niche in the market for new brands" its strengthening the conclusion that there is no other way to increase market share
(C) Two of the three mergers in the industry's last ten years led to a decline in profits and revenues for the the newly merged companies. -Argument talking about market share, not profits and revenues.
(D) Price reductions, achieved by cost-cutting in production, by some of the dominant firms in the carpet market are causing other producers to leave the market altogether. -Correct. Weakens the conclusion that there is no other way to increase market share. If other producers are leaving due to some other reason, then automatically market share for the existing ones will increase
(E) The carpet market is unlike most markets in that consumers are becoming increasingly resistant to new patterns and styles. -Strengthening the argument. This means producers cannot increase market share by creating new patterns or designs
User avatar
Manager
Manager
Joined: 15 May 2009
Posts: 90
Own Kudos [?]: 71 [0]
Given Kudos: 3
 Q44  V44
Send PM
Re: There is relativley little room for growth in the overall [#permalink]
arvs212 wrote:
There is relativley little room for growth in the overall carpet market, which is tied to the size of the population. Most who purchase carpet do so only once or twice, first in their twenties ot thirties, and then perhaps again in their fifties or sixties. Thus as the population ages, companies producing carpet will be able to gain market share in the carpet market only through purchasing competitors, and not through more agressive marketing.

Which of the following, if true, casts the most doubt on the conclusion above?

(A) Most of the major carpet producers market other floor coverings as well.
(B) Most established carpet producers market several different brand names and varieties, and there is no remaining niche in the market for new brands to fill.
(C) Two of the three mergers in the industry's last ten years led to a decline in profits and revenues for the the newly merged companies.
(D) Price reductions, achieved by cost-cutting in production, by some of the dominant firms in the carpet market are causing other producers to leave the market altogether.
(E) The carpet market is unlike most markets in that consumers are becoming increasingly resistant to new patterns and styles.


Conclusion: the ONLY WAY to gain market share is to purchase competing companies, not by more aggressive marketing.
I think (D) undermines this conclusion, it demonstrates ways other than mergers (namely cost-cutting & price reduction) by which companies can gain market share. I know this does not undermine the second part of the conclusion, which is that "aggressive marketing cannot increase market share", but I think (D) is still the best of the available choices.

A - Irrelevant, does not strengthen or weaken.
B - This seems to strengthen, rather than weaken the conclusion. It reinforces the conclusion that "there is NO OTHER WAY" by eliminating a possible method of competition (i.e. exploiting niche markets).
C - This weakens somewhat, it gives examples of failures of the current method (namely, buying out the competition), but it does not undermine the conclusion that buy-outs are the ONLY WAY.
E - This seems to strengthen the conclusion due to the same reason as B, it eliminates a course of action, which, if successful, might weaken the conclusion.
User avatar
Intern
Intern
Joined: 01 May 2009
Posts: 32
Own Kudos [?]: 773 [0]
Given Kudos: 11
Send PM
Re: There is relativley little room for growth in the overall [#permalink]
D for me tooo.

When most of the guys are leaving the market there wont be any company to overtake.



Archived Topic
Hi there,
This topic has been closed and archived due to inactivity or violation of community quality standards. No more replies are possible here.
Where to now? Join ongoing discussions on thousands of quality questions in our Critical Reasoning (CR) Forum
Still interested in this question? Check out the "Best Topics" block above for a better discussion on this exact question, as well as several more related questions.
Thank you for understanding, and happy exploring!
GMAT Club Bot
Re: There is relativley little room for growth in the overall [#permalink]
Moderators:
GMAT Club Verbal Expert
6921 posts
GMAT Club Verbal Expert
238 posts

Powered by phpBB © phpBB Group | Emoji artwork provided by EmojiOne